for what is values of x does 4(3x-2) = 12x - 5? select all that apply.
A.12
B.-8
C.3
D.-3
E.none of these​

Answers

Answer 1

Answer:

‼️A) 12‼️

Step-by-step explanation:

Key skills needed: Interior Angle Measure Theorem, Equation Creation

Step 1) First, we need to classify this shape. It has 7 sides, which means that we can use the Interior angle Theorem to find out the sum of all the interior angle measures

The theorem is:    S = (n - 2)180S=(n−2)180

S is the sum of all interior angle measures

n is the number of sides of the polygon

Step 2) With this, we can plug in 7 for "n" (Since the figure has 7 sides) and get:

---> S = (7-2)180S=(7−2)180

---> (7-2) is 5 since 7 - 2 = 5 so --> S = 5(180)S=5(180)

---> 5(180) is the same as 5 x 180 which is 900 so --> S = 900S=900

This means that the sum of all interior angles is 900 degrees.

Step 3) Now, we have to find out the sum of all the angles that they gave us so:

---> 10x + 10x + 9 + 133 + 9x + 14 + 12x - 9 + 10x + 5 + 136 = 90010x+10x+9+133+9x+14+12x−9+10x+5+136=900

The left side is the sum of all interior angles in the shape, and the right side is what the sum should be when expressed as a number.

Step 4) We have to combine all like terms on the left side:

---> 10x + 10x + 9x + 12x + 10x = 51x10x+10x+9x+12x+10x=51x

---> 9 + 133 + 14 -9+5+136 = 2889+133+14−9+5+136=288    

Step 5) This means that ---> 51x + 288 = 90051x+288=900

Subtract 288 from both sides  and get:

---> 51x = 61251x=612    (900-288 = 612)

Then divide by 51 from both sides and get:

---> x = 12x=12     (612 ÷ 51 = 12)

Therefore A) 12 is your answer.

Answer 2

I think it should be none of these.

The term on the left has to be even no matter what value of x and the right has to be odd no matter what value of x. Therefore, there shouldn't be an integer value of x that satisfies the equation, let alone the answer choices A-D.


Related Questions

In an input/output table, all outputs are 0, regardless of the input. What could the function equation be? Select all that apply.

y = 0 x
y = x
y = x/0
y = 0/x

Answers

Answers:

choice A)   y = 0x

choice D)  y = 0/x

========================================================

Explanation:

Choice A is the same as y = 0 because 0x turns into 0. Multiplying 0 with any number always leads to 0.

Similarly, choice D is the same as y = 0 as well. Dividing 0 over any nonzero value leads to 0. Note the key term "nonzero" here. We cannot have 0 in the denominator. So x = 0 is not allowed for choice D.

So for any nonzero x, we have 0x and 0/x result in the same thing.

-----------

Extra info:

Choice B can be ruled out because something like x = 2 leads to y = 2.

Choice C is ruled out because we can never have 0 in the denominator.

please help me !! :)

Answers

Answer:

A

because x is the horizontal axis and it is moved forward by two because of the addition of 2

factorize. xy^2-y(x-z) -z​

Answers

Answer:

The equation x The equation x 2The equation x 2 +xy+xz+yz can be factorised as follows:The equation x 2 +xy+xz+yz can be factorised as follows:x The equation x 2 +xy+xz+yz can be factorised as follows:x 2The equation x 2 +xy+xz+yz can be factorised as follows:x 2 +xy+xz+yz=(x The equation x 2 +xy+xz+yz can be factorised as follows:x 2 +xy+xz+yz=(x 2The equation x 2 +xy+xz+yz can be factorised as follows:x 2 +xy+xz+yz=(x 2 +xy)+(xz+yz)=x(x+y)+z(x+y)=(x+z)(x+y)The equation x 2 +xy+xz+yz can be factorised as follows:x 2 +xy+xz+yz=(x 2 +xy)+(xz+yz)=x(x+y)+z(x+y)=(x+z)(x+y)Hence, x The equation x 2 +xy+xz+yz can be factorised as follows:x 2 +xy+xz+yz=(x 2 +xy)+(xz+yz)=x(x+y)+z(x+y)=(x+z)(x+y)Hence, x 2The equation x 2 +xy+xz+yz can be factorised as follows:x 2 +xy+xz+yz=(x 2 +xy)+(xz+yz)=x(x+y)+z(x+y)=(x+z)(x+y)Hence, x 2 +xy+xz+yz=(x+z)(x+y)

Answer:

= (y-1) (xy-z)

Step-by-step explanation:

First we expand them -

= xy^2 - xy - yz - z

= xy(y-1)-z(y-1)

= (y-1) (xy-z)

Hope this helped

Find the measure of

Answers

Answer:

<L is 90°

Step-by-step explanation:

all angles at the center are twice the angles at the circumference and the angle at the center is 180 meaning the angle at the circumference will be half of 180

I hope this helps

Evaluate each expression when a = 3, b = 12, c = 4.
1) Зас

Answers

Answer:

36

Step-by-step explanation:

Зас

Let a =3 and c = 4

3*3*4

9*4

36

5(15-7y/3)+2y=-4
solve for y and plz show steps​

Answers

Answer:

75 - 35y / 3 + 2y = -4

( 75 - 35y + 6y ) / 3 = -4 [ Taking LCM ]

( 75 - 29y ) / 3 = -4

75 - 29y = -12

29y = 75+12

29y = 87

y = 3

hope that helps ✌

I need this question

Answers

Can you translate cuss I can’t understand sorry

 If 2^x-4 = 4^x-6 , then value of x is?​

Answers

Answer: [tex]x=1[/tex]

Step-by-step explanation:

[tex]2^x-4=4^x-6[/tex] is the equation that you've given us.

Now if we plot these two equations on the graph we notice there's an intersection at (1,-2). Therefore meaning that [tex]x=1[/tex].

We can prove that by doing the following calculations to prove that both sides are equal to each other.

The left side of the equal sign:

Step 1: Write the equation down:

[tex]2^x-4[/tex]

Step 2: Substitute x for the numerical value we found.

[tex]2^1-4[/tex]

Step 3: Find the square of [tex]2^1[/tex], which is itself, 2.

[tex]2-4[/tex]

Step 4: Subtract 2 from 4. Which is a negative number, thus being -2.

[tex]-2[/tex]

The right side of the equal sign:

Step 1: Write the equation down:

[tex]4^x-6[/tex]

Step 2: Substitute x for the numerical value we found.

[tex]4^1-6[/tex]

Step 3: Find the square of [tex]4^1[/tex], which is itself, 4.

[tex]4-6[/tex]

Step 4: Subtract 4 from 6. Which is a negative number, thus being -2.

[tex]-2[/tex]

We know that [tex]x=1[/tex] because when substituting x with 1, we get -2 on both sides. Therefore making this statement true and valid.

[tex]-2=-2[/tex]

These circles are:

A. tangent
B. congruent
C. congruent
D. none of these

Answers

Answer:

A. tangent

Step-by-step explanation:

Hi there!

Tangent circles are circles that intersect (or "touch") at exactly one point. These circles appear to have this characteristic.

I hope this helps!

Yet another calculus question :)


Given [tex]y = x^3 - 2x[/tex] for [tex]x \geq 0[/tex], find the equation of the tangent line to y where the absolute value of the slope is minimized.

I have tried taking both the first and second derivatives and setting them equal to 0 and using that as the answer, but they're incorrect. Could somebody please explain how to complete the question correctly? Thank you so much!

Answers

Answer:  y = (-4/3)*sqrt(2/3)

This is the same as writing [tex]y = -\frac{4}{3}\sqrt{\frac{2}{3}}[/tex]

============================================================

Explanation:

The phrasing "where the absolute value of the slope is minimized" is an interesting way of saying "the tangent slope is 0". This is because absolute values are never negative, so the smallest it can get is 0.

Your teacher has given you

y = x^3 - 2x

which differentiates into

dy/dx = 3x^2 - 2

after using the power rule

The derivative function lets us determine the slope of the tangent. The slope is the dy/dx value. Since we want a slope of 0, we'll set 3x^2-2 equal to zero and solve for x. So you have the correct idea, but you won't involve the second derivative.

dy/dx = 0

3x^2 - 2 = 0

3x^2 = 2

x^2 = 2/3

x = sqrt(2/3)

Notice how I'm ignoring the negative version of this root. This is due to the fact that [tex]x \ge 0[/tex]

-------------------------

Now plug this x value back into the original equation to find its corresponding y coordinate.

y = x^3 - 2x

y = x(x^2 - 2)

y = sqrt(2/3)*( 2/3 - 2 )

y = sqrt(2/3)*( -4/3 )

y = (-4/3)*sqrt(2/3)

Note that x = sqrt(2/3) leads to x^2 = 2/3 after squaring both sides.

-------------------------

Therefore, the equation of this tangent line is y = (-4/3)*sqrt(2/3)

All horizontal lines are of the form y = k, for some constant k. This constant value is basically what number you want the horizontal line to go through on the y axis. That number would be (-4/3)*sqrt(2/3).

In the following diagram, BAC = 46 degrees, and AD is the bisector of BAC. Find BDA

Answers

Answer:

67 degrees

Step-by-step explanation:

We have a right triangle with the following known values:

B = 90

A = 23 (46/2=23 since AD is the angle bisector)

D = ?

180 = 90 + 23 + D and we solve

67 = D

what is the equation of the axis of symmetry do the quadratic function f(x) = -(x+4) (x-1)​

Answers

9514 1404 393

Answer:

  x = -3/2

Step-by-step explanation:

The zeros of the function are the values of x that make the factors zero:

  x = -4, x = 1

The axis of symmetry is the vertical line halfway between these zeros.

  x = (-4 +1)/2 = -3/2

The equation of the axis of symmetry is x = -3/2.

find the missing length indicated​

Answers

Answer:

192

Step-by-step explanation:

According to Euclidian theorem

x^2 = 144*(400-144)

x^2 = 144*256

x^2 = 36864 find the root of both

x = 192

Density of wood is a very important trait for furniture making. There are two main important mechanical properties: strength and stiffness. Research has shown that a increase density equals an increase strength, hardness and stiffness of sawn timber.


A carpenter has two pieces of wood. He wants to find the density of each piece.


The first piece came from a birch tree. The piece of birch wood measures 2 meters by 4 meters by 3 meters and weighs (mass) 16 kilograms.


The second piece came from a cypress tree. The piece of cypress wood measures 3 meters by 5 meters by 2 meters and weighs (mass) 15.3 kilograms.


Which piece of wood has a greater density? You must show your work in order to receive credit.

Answers

Step-by-step explanation:

The Birch tree has a greater density.


Work out an estimate for the value of
41.2 x 19.8
——————-
0.49

Answers

Answer:

820

Step-by-step explanation:

41.2 * 19.8 = 41 *20 = 820

41.2

In this tenth value is 2 which is less than 5. So ignore it and write the whole number alone. 41

19.8

In this tenth value is 8 awhich greater than 5. So add 1 to whole part and write.

19+1 = 20

Provide a rule to describe the relationship between the number in the sequence

1,4,9,16​

Answers

Answer:

add 3, add 5, add 3

Step-by-step explanation:

1, 4, 9, 12

add 3, add 5, add 3

Answer:

x^2

Step-by-step explanation:

1^2 = 1

2^2 = 4

3^2 = 9

4^2 = 16

And so on.

So, the rule is x^2.

factor: 3x^(2)-16x+16

Answers

Answer:

[tex]3 {x}^{2} - 16x + 16 \\ 3 {x}^{2} - 12x - 4x + 16 \\ 3x(x - 4) - 4(x - 4) \\ (3x - 4)(x - 4)[/tex]

Step-by-step explanation:

I hope it helped U

stay safe stay happy

Answer is (3x-4) (x-4)
This is the answer for Plato and Edmentum

31. Which choice describes the value of m when -5(m + 1) = 23?
А
B.
m 2-3
ms-
6 m2
D.ms-

Answers

Answer:

The correct choice is Option A. m ≥ -28/5

Step-by-step explanation:

Find the 8th term of the geometric sequence 7,−21,63,

Answers

Answer:

8th term is -15309

Step-by-step explanation:

[tex]{ \boxed{ \bf{u_{n} = a( {r}^{n - 1} ) }}} \\ { \tt{u_{8} = 7( {( - 3)}^{8 - 1}) }} \\ { \tt{u_{8} = 7( - 2187)}} \\ { \tt{u _{8} = - 15309}}[/tex]

r is the common difference, r = -21/7 = -3

Answer:

a₈ = - 15309

Step-by-step explanation:

The nth term of a geometric sequence is

[tex]a_{n}[/tex] = a₁ [tex](r)^{n-1}[/tex]

where a₁ is the first term and r the common ratio

Here a₁ = 7 and r = [tex]\frac{a_{2} }{a_{1} }[/tex] = [tex]\frac{-21}{7}[/tex] = - 3 , then

a₈ = 7 × [tex](-3)^{7}[/tex] = 7 × - 2187 = - 15309

1 upon sec A -tan a minus one upon cos A is equal to 1 upon Cos A minus one upon sec a + tan a prove it​

Answers

Step-by-step explanation:

1/(seca-tana) - 1/cosa=1/cosa -1/(seca+tana)

There are many ways to solve this, let's take a look at a simple one

1/(seca-tana) + 1/(seca+tana) =1/cosa + 1/cosa

((seca-tana)+(seca+tana))/(sec^2a-tan^2a)=2/cosa

(seca-tana+seca+tana)/1=2*1/cosa

2seca=2seca

Please help me this on the picture

Answers

Answer:

Step-by-step explanation:

125 + (a-b)³ help me​

Answers

Answer:

(5 + a - b)(a² + b² - 2ab - 5a + 5b + 25)

Step-by-step explanation:

125 + (a - b)³ ← is a sum of cubes and factors in general as

a³ + b³ = (a + b)(a² - ab + b²)

Then

125 + (a- b)³

= 5³ + (a - b)³

with a = 5 and b = (a - b)

= (5 a a - b)(5² - 5(a - b) + (a - b)² )

= (5 + a - b)(25 - 5a + 5b + a² - 2ab + b²)

= (5 + a - b)(a² + b² - 2ab - 5a + 5b + 25)

Hey can someone help me figure out what 3/4(x-8)=12

Answers

Answer:

x=24

Step-by-step explanation:

3/4(x-8)=12

Multiply each side by 4/3

4/3*3/4(x-8)=12*4/3

x-8 = 16

Add 8 to each side

x-8+8 = 16+8

x=24

Answer:

x = 24

Step-by-step explanation:

Essentially, you need to break this apart.

You need to isolate the x to figure it out with division or something.

To isolate, you need to get rid of the numbers around it.

To get rid of the 3/4, you can convert it to a decimal to make it easier, and then divide both sides by 0.75 to get rid of it on the x's side.

You now have

x-8 = 12/0.75

You can quickly punch the right side into a calculator to make it easier.

x - 8 = 16

You then add 8 to both sides to finish off the x, to get

x = 24

pls help !! answer both 1a and 1b pls​

Answers

Answer:

KM = 50 mm , ∠ K = 104°

Step-by-step explanation:

Note that KM = KL = 50mm ( congruent legs )

This indicates the triangle is isosceles with base angles congruent, both 38°

∠ K = 180° - (38 + 38)° = 180° - 76° = 104°

Answer:

So the answer is 398°

Step-by-step explanation:

hope it helps

Setting y = 0 allows you to determine the what
of a graph

Answers

This allows you to find the x intercepts of the function.

  [tex]y=0[/tex] allows you to determine the x-intercept of a graph.

x-intercept and y-intercept:The points where a line crosses an axis are known as the x-intercept and the y-intercept, respectively.By changing Y to 0 in the equation and figuring out X, you can always determine the X-intercept. Similarly, by putting X to 0 in the equation and solving for Y, you can always determine the Y-intercept.When y is 0, the x-intercept is reached. The graph's intersection with the y-axis, or (0, b), is known as the y-intercept. When x is 0, the y-intercept is reached. When y is 0, the x-intercept is reached.

Therefore, [tex]y=0[/tex] allows you to determine the x-intercept of a graph.

Know more about x-intercept and y-intercept here:

https://brainly.com/question/24363347

#SPJ2

Simplify [2(5 – 4) + 6(7 – 3)] ÷ 2

I'LL MARK BRAINLY PLEASE HELP ME I'M STRUGGLING

Answers

Answer:

13

Step-by-step explanation:

[2×1+6×4]÷2

[2+24]÷2

26÷2

13

Please help explanation if possible

Answers

Answer:

y=-3x+5

Step-by-step explanation:

concepts: y=mx+b is slope intercept equation formula

m=slope

b= y intercept

Therefore we need to find the slope and y intercept

first find slope

m=[tex]\frac{y2-y1}{x2-x1}[/tex]

let y2 be 2

let y1 be -4

let x2 be 1

let x1 be 3

[tex]\frac{2-(--4)}{1-3}[/tex]

6/-2 = -3

m=-3

slope is -3

y= -3x+ b

we need find y intercept now

just plug in (1,2) into that equation

2=-3(1)+b

b=5

y=-3x+5

Which regular polygon can be used to form a tessellation?


I need help

Answers

Answer: Option 4 (Hexagon)

Step-by-step explanation:

A equilateral triangle, square and hexagon can be used to form a tessellation.

Please click thanks and mark brainliest if you like

10,4,7
The above set 4 numbers have a mean of 8
what could the mystery number

Answers

11 is the mystery number

Answer:

The mystery number is 11

Step-by-step explanation:

→ Set up an equation

( 10 + 4 + 7 + x ) ÷ 4 = 8

→ Multiply both sides by 4

21 + x = 32

→ Minus 21 from both sides

x = 11

solve this questions​

Answers

Answer:

3 a) solution:-

Discount=Rs 1,200

S .P =RS 4,000

M P=?

we know that,

Discount= M.P - S.P

or, Rs 1,200= M.P - Rs 4,000

or, M.P = RS (4,000-1,200 )

M.P = RS 2,800

In no. 3 all we have to make like this .........

Hope it helps

Answer:

Step-by-step explanation:

2)a) M.P =₹ 5000

S.P = ₹4000

Discount amount = M.P - S.P

                            = 5000 - 4000

                            = ₹1000

Discount percent = [tex]\frac{discount \ amount}{M.P}*100[/tex]

                             [tex]= \frac{1000}{5000}*100\\[/tex]

                             = 20%

3)a) Discount = ₹ 1200

S.P = ₹ 4000

M.P = S.P + discount

      = 4000 + 1200

      =  ₹ 5200

4)a ) Discount rate = 15%

S.P= ₹ 4000

[tex]M.P = \frac{100+Discount rate}{100}*S.P\\\\=\frac{115}{100}*4000[/tex]

= ₹ 4600

                           

                         

Other Questions
In a jail cell, there are 5 Democrats and 6 Republicans. Four of these people will bechosen for early release. How many 4-person groups are possible? please help i will give brainliest (check both images) WILL AWARD BRANLIEST !!!Triangle JKL was dilated with the origin as the center of dilation to create triangle J'K'L'. The triangle was dilated using a scale factor of 1/2.The lengths of the sides of triangle JKL are given.Enter the lengths of the sides of triangle J'K'L' below.(Decimal values may be used.)JK=6 unitsJ'K'= _unitsKL=10.5 unitsK'L'= _unitsLJ=12 unitsL'J'= _ units You saw that some words were on the portrait. How did you interpret the words on the painting? How is interpreting words on an image different from interpreting words on a page? Make a documentary on the Journey of Coronavirus OR Conservation of Plants and Animals. Give an appropriate name for your documentary. PLSS REPLY FAST A problem facing American iron and steel companies Shelley drove from New Haven, Connecticut, to New York City in 90 minutes. Which equation relates the distance she traveled to her speed? a.distance = speed + 90b.distance = speed 90c.distance speed = 90d.distance 90 = speed What did the Kansas-Nebraska Act give voters in the Kansas and Nebraskaterritories the right to do? How does Dickens use language to prove that the Master is in great shock when Oliver asks for more? PLEASEEEE HELP, how do i draw this ? A lottery has a grand prize of 25and 1/2 million dollars. There are 5 people who have winning tickets and they share this prize equally. How many millions of dollars does each person get? Express the answer as a mixed number. It is not possible to achieve 100% quality of life, why? Give reason. Look at the numbers below. 9.8 5.4 1.0 14.8 Which shows the best way to add these numbers using the Commutative and Associative Properties? A. (9.8 + 1.0) + (5.4 + 14.8) B. (9.8 + 14.8) + (5.4 + 1.0) C. (1.0 + 14.8) + (9.8 + (5.4)) D. (1.0 + (9.8)) + (14.8 + (5.4) PLEASE HELP! much appreciated :DFind the value of x. which of the following statement about carbon dixoide is not true?a.) increased carbon dioxide in the atmosphere increases the green house effectb.) carbon dioxide is a greenhouse gasc.) atmospheric carbon dioxide levels have slowly declined over the last 50 yearsd.) burning fossil fuels can increase carbon dioxide levels in the atmosphere What is the percent yield of a reaction?O A. The difference between measured and calculated amountsOB.The amount of produd obtainedamount possible* 100C. The amount of product actually obtained in a reactionD. The amount of product that is possible from a reaction In how many ways can alice give 12 apples to 3 children if each child must get at least 1 apple What is Soviet Union. when and why was it formed?plz help which of the following is an example of an arrthenius acid The burden of a tax is shared by producers and consumers. Under what conditions will consumers pay most of the tax? Under what conditions will producers pay most of it? A. It depends on who is legally obligated to pay the tax. Typically producers are required to pay the tax and therefore bear most of the burden. B. If demand is relatively more elastic than supply, producers will pay more of the tax. C. It depends on who is legally obligated to pay the tax. Typically consumers are required to pay the tax and therefore bear most of the burden. D. If demand is relatively less elastic than supply, producers will pay more of the tax.